You are on page 1of 16

AP US GoPo - Supreme Court Cases to Know 2013-2014

Case Year Case Overview


Question - Is the execution of mentally retarded persons "cruel and unusual punishment" prohibited by the
Atkins v. Virginia 2001 Eighth Amendment?
Daryl Atkins convicted of abduction, armed robbery and murder with the only witness being a
psychologist who deemed him mentally retarded. In a second trial in VA which found him guilty again
used precedence in Penry v. Lynaugh that no capital punishment for the mentally disabled.
Question – Did the SC have jurisdiction over questions of legislative apportionment? Charles Baker
Baker v Carr 1962 claimed that a 1901 law to apportion seats in TN General Assembly ignored sufficient economic growth,
and population shifts in the State. Court ruled for Baker and 14th Amendment’s equal protection and
needed judicial evaluation.
Question – Does the 5th Amendment deny the states as well as the national government the right to take
Barron v. Baltimore 1833 private property for public use without justly compensating the property’s owner?
John Barron was co-owner of a profitable wharf in the harbor of Baltimore. As the city developed and
expanded, large amounts of sand accumulated in the harbor, depriving Barron of the deep waters which
had been the key to his successful business. He sued the city to recover a portion of his financial losses.
The Court announced its decision in this case without even hearing the arguments of the City of Baltimore.
Writing for the unanimous Court, Chief Justice Marshall found that the limitations on government
articulated in the Fifth Amendment were specifically intended to limit the powers of the national
government. Citing the intent of the framers and the development of the Bill of Rights as an exclusive
check on the government in Washington D.C., Marshall argued that the Supreme Court had no jurisdiction
in this case since the Fifth Amendment was not applicable to the states.
Question - Did the Board of Education's decision to ban certain books from its junior high and high school
Board of Ed (Island Tree) 1981 libraries, based on their content, violate the First Amendment's freedom of speech protections?
v. Pico The Island Trees Union Free School District's Board of Education (the "Board"), acting contrary to the
recommendations of a committee of parents and school staff, ordered that certain books be removed from
its district's junior high and high school libraries. In support of its actions, the Board said such books were:
"anti-American, anti-Christian, anti-Semitic, and just plain filthy." Acting through his friend Francis Pico,
and on behalf of several other students, Steven Pico brought suit in federal district court challenging the
Board's decision to remove the books. The Board won; the U.S. Court of Appeals for the Second Circuit
reversed. The Board petitioned the U.S. Supreme Court, which granted certiorari.
Although school boards have a vested interest in promoting respect for social, moral, and political
community values, their discretionary power is secondary to the transcendent imperatives of the First
Amendment. the Board could not restrict the availability of books in its libraries simply because its
members disagreed with their idea content.
Question - Does the Constitution confer a fundamental right upon homosexuals to engage in consensual
Bowers v. Hardwick 1986 sodomy, thereby invalidating the laws of many states which make such conduct illegal?
Michael Hardwick was observed by a Georgia police officer while engaging in the act of consensual
homosexual sodomy with another adult in the bedroom of his home. After being charged with violating a
Georgia statute that criminalized sodomy, Hardwick challenged the statute's constitutionality in Federal
District Court.
The divided Court found that there was no constitutional protection for acts of sodomy, and that states
could outlaw those practices. Justice White feared that guaranteeing a right to sodomy would be the
product of "judge-made constitutional law" and send the Court down the road of illegitimacy.
Question - Does the application of New Jersey's public accommodations law violate the Boy Scouts' First
Boy Scouts of America v. 1999 Amendment right of expressive association to bar homosexuals from serving as troop leaders?
Dale Boy Scouts of America revoked former Eagle Scout and assistant scoutmaster James Dale's adult
membership when the organization discovered that Dale was a homosexual and a gay rights activist. In
1992, Dale filed suit against the Boy Scouts, alleging that the Boy Scouts had violated the New Jersey
statute prohibiting discrimination on the basis of sexual orientation in places of public accommodation.
The Boy Scouts, a private, not-for-profit organization, asserted that homosexual conduct was inconsistent
with the values it was attempting to instill in young people. The New Jersey Superior Court held that New
Jersey's public accommodations law was inapplicable because the Boy Scouts was not a place of public
accommodation. The court also concluded that the Boy Scouts' First Amendment freedom of expressive
association prevented the government from forcing the Boy Scouts to accept Dale as an adult leader. The
court's Appellate Division held that New Jersey's public accommodations law applied to the Boy Scouts
because of its broad-based membership solicitation and its connections with various public entities, and
that the Boy Scouts violated it by revoking Dale's membership based on his homosexuality. The court
rejected the Boy Scouts' federal constitutional claims. The New Jersey Supreme Court affirmed. The court
held that application of New Jersey's public accommodations law did not violate the Boy Scouts' First
Amendment right of expressive association because Dale's inclusion would not significantly affect
members' abilities to carry out their purpose. Furthermore, the court concluded that reinstating Dale did
not compel the Boy Scouts to express any message.
CJ Rehnquist, the Court held that "applying New Jersey's public accommodations law to require the Boy
Scouts to admit Dale violates the Boy Scouts' First Amendment right of expressive association." In effect,
the ruling gives the Boy Scouts of America a constitutional right to bar homosexuals from serving as troop
leaders. Chief Justice Rehnquist wrote for the Court that, "[t]he Boy Scouts asserts that homosexual
conduct is inconsistent with the values it seeks to instill," and that a gay troop leader's presence "would, at
the very least, force the organization to send a message, both to the young members and the world, that the
Boy Scouts accepts homosexual conduct as a legitimate form of behavior."
Question - Did the questioning and utilization of the so-called “Christian burial speech” constitute an
Brewer v. Williams 1977 interrogation in which the defendant was entitled to counsel, such that the absence of counsel deprived him
of his Sixth Amendment rights? Based on the facts presented, did defendant waive his right to counsel?
Mr. Williams was arrested for abducting a ten-year-old girl in Des Moines, Iowa. Williams was
represented by counsel at his arraignment in Davenport, Iowa, and had another attorney in Des
Moines. Police Officers transporting Williams knew that he was represented by counsel and agreed not to
question him during the trip. Despite that acknowledgment, officers repeatedly questioned Williams
during the drive. Mr. Williams initially exhibited no desire to speak with officers, informing them that he
would talk to them only after conferring with his attorney. However, one of the officers knew that Mr.
Williams was a deeply religious former mental patient, and suggested that it was important for the police
to locate the missing girl’s body so that her parents could provide a decent Christian burial. Thereafter,
Mr. Williams made incriminating statements and eventually lead police to the girl’s body. Tried and
convicted of murder, Williams argued that the incriminating statements were improperly obtained in
violation of his Sixth Amendment right to counsel. The Iowa Supreme Court affirmed his
conviction. This case came to the Court through a habeas petition following exhaustion of appellate
remedies in state court. Mr. Brewer, as superintendent of the facility holding Williams, was named as the
defendant in that action.
Justice Stewart delivered the opinion of the Court, concluded that officers had violated the defendant’s
rights. First, the majority explained that while the district court in this habeas proceeding had made
additional findings of fact based on state court records, they had carefully explained such findings and had
not disputed the factual determinations of the state court in violation of federal law. Next, the Court
concluded it was unnecessary to evaluate whether statement in question was voluntary, or whether the
statement violated protections against self-incrimination, focusing instead on the right to counsel
guaranteed by the Sixth and Fourteenth Amendments. In that analysis, the Court reasoned that the
“Christian Burial Speech” was tantamount to an interrogation, designed to illicit responses just as surely as
any other form of interrogation.
Equal protection – Linda Brown was denied admission to a public school on the basis that she was black.
Brown v. Board of Ed. 1954 Chief Justice Earl Warren overruled “separate but Equal” in Plessy and he stated that public schools
violated the 14th amendment of equal protection. Court decision declared that segregation must end.
Question - Did the limits placed on electoral expenditures by the Federal Election Campaign Act of 1971,
Buckley v. Valeo 1977 and related provisions of the Internal Revenue Code of 1954, violate the First Amendment's freedom of
speech and association clauses?
In the wake of the Watergate affair, Congress attempted to ferret out corruption in political campaigns by
restricting financial contributions to candidates. Among other things, the law set limits on the amount of
money an individual could contribute to a single campaign and it required reporting of contributions above
a certain threshold amount. The Federal Election Commission was created to enforce the statute.
Court arrived at two important conclusions. First, it held that restrictions on individual contributions to
political campaigns and candidates did not violate the First Amendment since the limitations of the FECA
enhance the "integrity of our system of representative democracy" by guarding against unscrupulous
practices. Second, the Court found that governmental restriction of independent expenditures in
campaigns, the limitation on expenditures by candidates from their own personal or family resources, and
the limitation on total campaign expenditures did violate the First Amendment. Since these practices do
not necessarily enhance the potential for corruption that individual contributions to candidates do, the
Court found that restricting them did not serve a government interest great enough to warrant a curtailment
on free speech and association.
Question - Did the Florida Supreme Court violate Article II Section 1 Clause 2 of the U.S. Constitution by
Bush v. Gore 2000 making new election law? Do standardless manual recounts violate the Equal Protection and Due Process
Clauses of the Constitution?
Following the U.S. Supreme Court's decision in Bush v. Palm Beach County Canvassing Board, and
concurrent with Vice President Al Gore's contest of the certification of Florida presidential election results,
on December 8, 2000 the Florida Supreme Court ordered that the Circuit Court in Leon County tabulate by
hand 9000 contested ballots from Miami-Dade County. It also ordered that every county in Florida must
immediately begin manually recounting all "under-votes" (ballots which did not indicate a vote for
president) because there were enough contested ballots to place the outcome of the election in doubt.
Governor George Bush and his running mate, Richard Cheney, filed a request for review in the U.S.
Supreme Court and sought an emergency petition for a stay of the Florida Supreme Court's decision. The
U.S. Supreme Court granted review and issued the stay on December 9. It heard oral argument two days
later.
Noting that the Equal Protection clause guarantees individuals that their ballots cannot be devalued by
"later arbitrary and disparate treatment," the per curiam opinion held 7-2 that the Florida Supreme Court's
scheme for recounting ballots was unconstitutional. Rehnquist (in a concurring opinion joined by Scalia
and Thomas) argued that the recount scheme was also unconstitutional because the Florida Supreme
Court's decision made new election law, which only the state legislature may do. Breyer and Souter
(writing separately) agreed with the per curiam holding that the Florida Court's recount scheme violated
the Equal Protection Clause, but they dissented with respect to the remedy, believing that a constitutional
recount could be fashioned. Time is insubstantial when constitutional rights are at stake. Ginsburg and
Stevens (writing separately) argued that for reasons of federalism, the Florida Supreme Court's decision
ought to be respected. Moreover, the Florida decision was fundamentally right; the Constitution requires
that every vote be counted.

Question - Does the application of the statute violate Chaplinsky's freedom of speech protected by the First
Chaplinsky v. NH 1941 Amendment?
Chaplinsky, a Jehovah's Witness, called a city marshal a "God-damned racketeer" and "a damned fascist"
in a public place. He was arrested and convicted under a state law for violating a breach of the peace.
Some forms of expression--among them obscenity and fighting words--do not convey ideas and thus are
not subject to First Amendment protection. In this case, Chaplinsky uttered fighting words, i.e., words that
"inflict injury or tend to incite an immediate breach of the peace."
Question - 1) Did the Supreme Court's decision in McConnell resolve all constitutional as-applied
Citizens United v. F.E.C. 2008 challenges to the BCRA when it upheld the disclosure requirements of the statute as constitutional?
2) Do the BCRA's disclosure requirements impose an unconstitutional burden when applied to
electioneering requirements because they are protected "political speech" and not subject to regulation as
"campaign speech"?
3) If a communication lacks a clear plea to vote for or against a particular candidate, is it subject to
regulation under the BCRA?
4) Should a feature length documentary about a candidate for political office be treated like the
advertisements at issue in McConnell and therefore be subject to regulation under the BCRA?
Citizens United sought an injunction against the Federal Election Commission in the United States District
Court for the District of Columbia to prevent the application of the Bipartisan Campaign Reform Act
(BCRA) to its film Hillary: The Movie. The Movieexpressed opinions about whether Senator Hillary
Rodham Clinton would make a good president.
In an attempt to regulate "big money" campaign contributions, the BCRA applies a variety of restrictions
to "electioneering communications." Section 203 of the BCRA prevents corporations or labor unions from
funding such communication from their general treasuries. Sections 201 and 311 require the disclosure of
donors to such communication and a disclaimer when the communication is not authorized by the
candidate it intends to support.
Citizens United argued that: 1) Section 203 violates the First Amendment on its face and when applied
to The Movie and its related advertisements, and that 2) Sections 201 and 203 are also unconstitutional as
applied to the circumstances.
The United States District Court denied the injunction. Section 203 on its face was not unconstitutional
because the Supreme Court in McConnell v. FEC had already reached that determination. The District
Court also held that The Movie was the functional equivalent of express advocacy, as it attempted to
inform voters that Senator Clinton was unfit for office, and thus Section 203 was not unconstitutionally
applied. Lastly, it held that Sections 201 and 203 were not unconstitutional as applied to the The Movie or
its advertisements. The court reasoned that the McConnell decision recognized that disclosure of donors
"might be unconstitutional if it imposed an unconstitutional burden on the freedom to associate in support
of a particular cause," but those circumstances did not exist in Citizen United's claim.

By a 5-to-4 vote along ideological lines, the majority held that under the First Amendment corporate
funding of independent political broadcasts in candidate elections cannot be limited. Justice Anthony M.
Kennedy wrote for the majority joined by Chief Justice John G. Roberts and Justices Antonin G. Scalia,
Samuel A. Alito, and Clarence Thomas. Justice John Paul Stevens dissented, joined by Justices Ruth
Bader Ginsburg, Stephen G. Breyer, and Sonia Sotamayor. The majority maintained that political speech is
indispensable to a democracy, which is no less true because the speech comes from a corporation. The
majority also held that the BCRA's disclosure requirements as applied toThe Movie were constitutional,
reasoning that disclosure is justified by a "governmental interest" in providing the "electorate with
information" about election-related spending resources. The Court also upheld the disclosure requirements
for political advertising sponsors and it upheld the ban on direct contributions to candidates from
corporations and unions.
In a separate concurring opinion, Chief Justice Roberts, joined by Justice Alito, emphasized the care with
which the Court handles constitutional issues and its attempts to avoid constitutional issues when at all
possible. Here, the Court had no narrower grounds upon which to rule, except to handle the First
Amendment issues embodied within the case. Justice Scalia also wrote a separate concurring opinion,
joined by Justices Alito and Thomas in part, criticizing Justice Stevens' understanding of the Framer's view
towards corporations. Justice Stevens argued that corporations are not members of society and that there
are compelling governmental interests to curb corporations' ability to spend money during local and
national elections.

Question - Did the President's ability to selectively cancel individual portions of bills, under the Line Item
Clinton v. NY 1997 Veto Act, violate the Presentment Clause of Article I?
This case consolidates two separate challenges to the constitutionality of two cancellations, made by
President William J. Clinton, under the Line Item Veto Act ("Act"). In the first, the City of New York, two
hospital associations, a hospital, and two health care unions, challenged the President's cancellation of a
provision in the Balanced Budget Act of 1997 which relinquished the Federal Government's ability to
recoup nearly $2.6 billion in taxes levied against Medicaid providers by the State of New York. In the
second, the Snake River farmer's cooperative and one of its individual members challenged the President's
cancellation of a provision of the Taxpayer Relief Act of 1997. The provision permitted some food refiners
and processors to defer recognition of their capital gains in exchange for selling their stock to eligible
farmers' cooperatives. After a district court held the Act unconstitutional, the Supreme Court granted
certiorari on expedited appeal.
In a 6-to-3 decision the Court first established that both the City of New York, and its affiliates, and the
farmers' cooperative suffered sufficiently immediate and concrete injuries to sustain their standing to
challenge the President's actions. The Court then explained that under the Presentment Clause, legislation
that passes both Houses of Congress must either be entirely approved (i.e. signed) or rejected (i.e. vetoed)
by the President. The Court held that by canceling only selected portions of the bills at issue, under
authority granted him by the Act, the President in effect "amended" the laws before him. Such discretion,
the Court concluded, violated the "finely wrought" legislative procedures of Article I as envisioned by the
Framers.
Question - Can a state deny unemployment benefits to a worker fired for using illegal drugs for religious
Dept of Human Resources 1987 purposes?
of Oregon v. Smith Two Native Americans who worked as counselors for a private drug rehabilitation organization, ingested
peyote -- a powerful hallucinogen -- as part of their religious ceremonies as members of the Native
American Church. As a result of this conduct, the rehabilitation organization fired the counselors. The
counselors filed a claim for unemployment compensation. The government denied them benefits because
the reason for their dismissal was considered work-related "misconduct."
Yes. Justice Antonin Scalia, writing for the majority, observed that the Court has never held that an
individual's religious beliefs excuse him from compliance with an otherwise valid law prohibiting conduct
that government is free to regulate. Allowing exceptions to every state law or regulation affecting religion
"would open the prospect of constitutionally required exemptions from civic obligations of almost every
conceivable kind." Scalia cited as examples compulsory military service, payment of taxes, vaccination
requirements, and child-neglect laws.
Question - Whether provisions of the D.C. Code generally barring the registration of handguns, prohibiting
District of Columbia v. 2007 carrying a pistol without a license, and requiring all lawful firearms to be kept unloaded and either
Heller disassembled or trigger locked violate the Second Amendment rights of individuals who are not affiliated
with any state-regulated militia, but who wish to keep handguns and other firearms for private use in their
homes?
For the first time in seventy years, the Court heard a case regarding the central meaning of the Second
Amendment and its relation to gun control laws. After the District of Columbia passed legislation barring
the registration of handguns, requiring licenses for all pistols, and mandating that all legal firearms must be
kept unloaded and disassembled or trigger locked, a group of private gun-owners brought suit claiming the
laws violated their Second Amendment right to bear arms. The federal trial court in Washington D.C.
refused to grant the plaintiffs relief, holding that the Second Amendment applies only to militias, such as
the National Guard, and not to private gun ownership.
The U.S. Court of Appeals for the District of Columbia Circuit disagreed, voting two to one that the
Second Amendment does in fact protect private gun owners such as plaintiffs. Petitioners agree with the
trial court's decision that the Second Amendment applies only to militias, and further argue that (a) the
Second Amendment should not apply to D.C. because it is a federal enclave rather than a state, and (b) that
the D.C. legislation merely regulates, rather than prohibits, gun ownership. Respondents, although
disagreeing on the merits, have also urged the Court to review the case in order to clearly define the
relationship between federal gun control laws and the Second Amendment.
In a 5-4 decision, the Court held that the Second Amendment protects an individual right to possess a
firearm unconnected with service in a militia, and to use that firearm for traditionally lawful purposes,
such as self- defense within the home. The Court based its holding on the text of the Second Amendment,
as well as applicable language in state constitutions adopted soon after the Second Amendment. Justice
Antonin Scalia delivered the opinion of the Court. Justices John Paul Stevens and Stephen Breyer filed
dissenting opinions, each joined by the other as well as Justices David Souter and Ruth Bader Ginsburg.
Justice Stevens argued that the Second Amendment only protects the rights of individuals to bear arms as
part of a well-regulated state militia, not for other purposes even if they are lawful. Justice Breyer agreed
with Stevens' argument but also stated that even if possession were to be allowed for other reasons, any
law regulating the use of firearms would have to be "unreasonable or inappropriate" to violate the Second
Amendment. In Breyer's view, the D.C. laws at issue in this case were both reasonable and appropriate.

Question - Did the city's conditions for the permit violate the 5th Amendment's "takings" clause as
Dolan v. City of Tigard 1993 absorbed by the 14th Amendment's due process clause?
Florence Dolan wanted a permit from the City of Tigard to expand her store and pave her parking lot. The
city agreed to grant her permit on the condition that she dedicate part of her land for (1) a greenway along
a nearby creek to help alleviate runoff from the pavement, and (2) a pedestrian/bicycle path to relieve
traffic congestion from the city's growing business district.
The Court ruled that the city did not present conclusive evidence that the walkway/bicycle path would
reduce traffic congestion, and so could not require Dolan to give up her property as a condition of the
permit. In addition, the city did not explain why a public greenway was necessary, as opposed to a private
one. There must be an "essential nexus" between a legitimate state interest and the permit requirements
(Nollan v. California Coastal Commission), and the city failed to demonstrate that the benefits would
justify the requirements.
Question - Did the Louisiana law, which mandated the teaching of "creation science" along with the theory
Edwards v. Aguillard 1986 of evolution, violate the Establishment Clause of the First Amendment as applied to the states through the
Fourteenth Amendment?
A Louisiana law entitled the "Balanced Treatment for Creation-Science and Evolution-Science in Public
School Instruction Act" prohibited the teaching of the theory of evolution in the public schools unless that
instruction was accompanied by the teaching of creation science, a Biblical belief that advanced forms of
life appeared abruptly on Earth. Schools were not forced to teach creation science. However, if either topic
was to be addressed, evolution or creation, teachers were obligated to discuss the other as well.
The Court held that the law violated the Constitution. Using the three- pronged test that the Court had
developed in Lemon v. Kurtzman (1971) to evaluate potential violations of the Establishment Clause,
Justice Brennan argued that Louisiana's law failed on all three prongs of the test. First, it was not enacted
to further a clear secular purpose. Second, the primary effect of the law was to advance the viewpoint that
a "supernatural being created humankind," a doctrine central to the dogmas of certain religious
denominations. Third, the law significantly entangled the interests of church and state by seeking "the
symbolic and financial support of government to achieve a religious purpose."
Question - Does Michael Newdow have standing to challenge as unconstitutional a public school district
Elk Grove Unified School 2003 policy that requires teachers to lead willing students in reciting the Pledge of Allegiance? Does a public
District v. Newdow school district policy that requires teachers to lead willing students in reciting the Pledge of Allegiance,
which includes the words "under God," violate the Establishment Clause of the First Amendment?
Michael Newdow's daughter attended public school in the Elk Grove Unified School District in California.
Elk Grove teachers began school days by leading students in a voluntary recitation of the Pledge of
Allegiance, including the words "under God" added by a 1954 Congressional act. Newdow sued in federal
district court in California, arguing that making students listen - even if they choose not to participate - to
the words "under God" violates the establishment clause of the U.S. Constitution's First Amendment.
The district court dismissed Newdow's complaint for lack of standing, because he and the mother of his
daughter are divorced and he does not have custody. The U.S. Ninth Circuit Court of Appeals reversed,
holding that Newdow did have standing "to challenge a practice that interferes with his right to direct the
religious education of his daughter." The Ninth Circuit ruled that Congress's 1954 act adding the words
"under God" to the Pledge and the school district policy requiring it be recited both violated the First
Amendment's establishment clause.
In an opinion authored by Justice John Paul Stevens, the Supreme Court found that Newdow did not have
standing to bring suit because he did not have sufficient custody over his daughter. "When hard questions
of domestic relations are sure to affect the outcome, the prudent course is for the federal court to stay its
hand rather than reach out to resolve a weighty question of federal constitutional law," Justice Stevens
wrote. Because it found that Newdow did not have standing, the Court failed to reach the constitutional
question. Chief Justice Renquist and Justices Sandra Day O'Connor and Clarence Thomas all wrote
seperate concurrences, saying that requiring teachers to lead the Pledge is constitutional.

Question - Does the reading of a nondenominational prayer at the start of the school day violate the
Engle v. Vitale 1962 "establishment of religion" clause of the First Amendment?
The Board of Regents for the State of New York authorized a short, voluntary prayer for recitation at the
start of each school day. This was an attempt to defuse the politically potent issue by taking it out of the
hands of local communities. The blandest of invocations read as follows: "Almighty God, we acknowledge
our dependence upon Thee, and beg Thy blessings upon us, our teachers, and our country."
SC ruled that public officials in NY could not require pupils to recite state composed prayer at the start of
every school day even if it was non-denominational and those who so desired could be excused.
Reason was that such official state sanction of religious utterances was an unconstitutional attempt to
establish religion. Neither the prayer's nondenominational character nor its voluntary character saves it
from unconstitutionality.
Question - Does a law forbidding the teaching of evolution violate either the free speech rights of teachers
Epperson v. Arkansas 1968 or the Establishment clause of the First Amendment?
The Arkansas legislature passed a law prohibiting teachers in public or state- supported schools from
teaching, or using textbooks that teach, human evolution. Epperson, a public school teacher, sued,
claiming the law violated her First Amendment right to free speech as well as the Establishment Clause.
The State Chancery Court ruled that it violated his free speech rights; the State Supreme Court reversed.
Seven members of the Court held that the statute violated the Establishment clause. Writing for the Court,
Justice Abe Fortas stated that the law had been based solely on the beliefs of fundamentalist Christians,
who felt that evolutionary theories directly contradicted the biblical account of Creation. This use of state
power to prohibit the teaching of material objectionable to a particular sect ammounted to an
unconstitutional Establishment of religion. Justice Fortas wrote, "The State's undoubted right to prescribe
the curriculum for its public schools does not carry with it the right to prohibit, on pain of criminal penalty,
the teaching of a scientific theory or doctrine where that prohibition is based upon reasons that violate the
First Amendment." The two other members of the Court concurred in the result, writing that it violated
either the Due Process clause of the 14th Amendment (because it was unconstitutionally vague) or the Free
Speech clause of the First Amendment.
Question - Could the contract between Fletcher and Peck be invalidated by an act of the Georgia
Fletcher v. Peck 1810 legislature?
Contracts and State Laws – first case to declare a state law in violation of the Constitution. Massive
bribery scandal involving GA’s legislature where it authorized the sale of thirty-five million acres to four
companies for less than 2 cents an acre and new legislature rescinded on the grant as purchases were
already selling off the land. When state rescinded, it broke the contract and revoked rights.
In a unanimous opinion, the Court held that since the estate had been legally "passed into the hands of a
purchaser for a valuable consideration," the Georgia legislature could not take away the land or invalidate
the contract. Noting that the Constitution did not permit bills of attainder or ex post facto laws, the Court
held that laws annulling contracts or grants made by previous legislative acts were constitutionally
impermissible.
Question - Does the imposition and carrying out of the death penalty in these cases constitute cruel and
Furman v. Georgia 1971 unusual punishment in violation of the Eighth and Fourteenth Amendments?
Furman was burglarizing a private home when a family member discovered him. He attempted to flee, and
in doing so tripped and fell. The gun that he was carrying went off and killed a resident of the home. He
was convicted of murder and sentenced to death (Two other death penalty cases were decided along with
Furman: Jackson v. Georgia and Branch v. Texas. These cases concern the constitutionality of the death
sentence for rape and murder convictions, respectively).
The Court's one-page per curiam opinion held that the imposition of the death penalty in these cases
constituted cruel and unusual punishment and violated the Constitution. In over two hundred pages of
concurrence and dissents, the justices articulated their views on this controversial subject. Only Justices
Brennan and Marshall believed the death penalty to be unconstitutional in all instances. Other
concurrences focused on the arbitrary nature with which death sentences have been imposed, often
indicating a racial bias against black defendants. The Court's decision forced states and the national
legislature to rethink their statutes for capital offenses to assure that the death penalty would not be
administered in a capricious or discriminatory manner.
Ferry dispute between NJ and NY – Aaron Ogden was given monopoly rights to Hudson by NY for ferry
Gibbons v. Ogden 1824 and US issued Thomas Gibbons rights to the same route. Case went to SC when Ogden sued Gibbons and
won – LANDMARK and established federal authority over the states.
Question - Does the Sixth Amendment's right to counsel in criminal cases extend to felony defendants in
Gideon v. Wainwright 1963 state courts?
Representation by Counsel – Gideon was accused of breaking into a poolroom in FL but had no financial
means to procure a defense which he requested and was refused and was forced to defend himself. The
court found him guilty and the SC ruled that people were entitled to legal counsel regardless of their ability
to pay for said counsel.
Question - Is the New York law punishing advocacy to overthrow the government by force an
Gitlow v. NY 1922 unconstitutional violation of the free speech clause of the First Amendment?
Gitlow, a socialist, was arrested for distributing copies of a "left-wing manifesto" that called for the
establishment of socialism through strikes and class action of any form. Gitlow was convicted under a
state criminal anarchy law, which punished advocating the overthrow of the government by force. At his
trial, Gitlow argued that since there was no resulting action flowing from the manifesto's publication, the
statute penalized utterences without propensity to incitement of concrete action. The New York courts had
decided that anyone who advocated the doctrine of violent revolution violated the law.
Threshold issue: Does the First Amendment apply to the states? Yes, by virtue of the liberty protected by
due process that no state shall deny (14th Amendment). On the merits, a state may forbid both speech and
publication if they have a tendency to result in action dangerous to public security, even though such
utterances create no clear and present danger. The rationale of the majority has sometimes been called the
"dangerous tendency" test. The legislature may decide that an entire class of speech is so dangerous that it
should be prohibited. Those legislative decisions will be upheld if not unreasonable, and the defendant will
be punished even if her speech created no danger at all.
Gonzales v. Oregon 2005 Question - Did the Controlled Substances Act authorize the attorney general to ban the use of controlled
substances for physician-assisted suicide in Oregon?
In 1994 Oregon enacted the Death with Dignity Act, the first state law authorizing physicians to prescribe
lethal doses of controlled substances to terminally ill patients. Attorney General John Ashcroft declared in
2001 that physician-assisted suicide violated the Controlled Substances Act of 1970 (CSA). Ashcroft
threatened to revoke the medical licenses of physicians who took part in the practice. Oregon sued
Ashcroft in federal district court. That court and, later the Ninth Circuit, held Ashcroft''s directive illegal.
The courts held that the CSA did not authorize the attorney general to regulate physician-assisted suicide,
which was the sort of medical matter historically entrusted to the states.
No. In a 6-3 opinion delivered by Justice Anthony Kennedy, the Court held that Congress intended the
CSA to prevent doctors only from engaging in illicit drug dealing, not to define general standards of state
medical practice. Moreover, the CSA did not authorize Attorney General John Ashcroft to declare a
medical practice authorized under state law to be illegitimate.
Question - Does the University of Michigan's use of racial preferences in undergraduate admissions violate
Gratz v. Bollinger 2002 the Equal Protection Clause of the Fourteenth Amendment or Title VI of the Civil Rights Act of 1964?
Grutter v. Bollinger n 1995, Jennifer Gratz applied to the University of Michigan's College of Literature, Science and the Arts
with an adjusted GPA of 3.8 and ACT score of 25. In 1997, Patrick Hamacher applied to the University
with an adjusted GPA of 3.0, and an ACT score of 28. Both were denied admission and attended other
schools. The University admits that it uses race as a factor in making admissions decisions because it
serves a "compelling interest in achieving diversity among its student body." In addition, the University
has a policy to admit virtually all qualified applicants who are members of one of three select racial
minority groups - African Americans, Hispanics, and Native Americans - that are considered to be
"underrepresented" on the campus. Concluding that diversity was a compelling interest, the District Court
held that the admissions policies for years 1995-1998 were not narrowly tailored, but that the policies in
effect in 1999 and 2000 were narrowly tailored. After the decision in Grutter, Gratz and Hamacher
petitioned the U.S. Supreme Court pursuant to Rule 11 for a writ of certiorari before judgment, which was
granted.
In a 6-3 opinion delivered by Chief Justice William H. Rehnquist, the Court held that the University of
Michigan's use of racial preferences in undergraduate admissions violates both the Equal Protection Clause
and Title VI. While rejecting the argument that diversity cannot constitute a compelling state interest, the
Court reasoned that the automatic distribution of 20 points, or one-fifth of the points needed to guarantee
admission, to every single "underrepresented minority" applicant solely because of race was not narrowly
tailored and did not provide the individualized consideration Justice Powell contemplated in Regents of the
University of California v. Bakke, 438 U.S. 265 (1978). Chief Justice Rehnquist wrote, "because the
University's use of race in its current freshman admissions policy is not narrowly tailored to achieve
respondents' asserted compelling interest in diversity, the admissions policy violates the Equal Protection
Clause."
The so-called Grandfather Clause of the amendment to the constitution of Oklahoma of 1910 is void
Guinn v. US 1913 because it violates the Fifteenth Amendment to the Constitution of the United States.
The Grandfather Clause being unconstitutional, and not being separable from the remainder of the
amendment to the constitution of Oklahoma of 1910, that amendment as a whole is invalid.
The Fifteenth Amendment does not, in a general sense, take from the States the power over suffrage
possessed by the States from the beginning, but it does restrict the power of the United States or the States
to abridge or deny the right of a citizen of the United States to vote on account of race, color or previous
condition of servitude. While the Fifteenth Amendment gives no right of suffrage, as its command is self-
executing, rights of suffrage may be enjoyed by reason of the striking out of discriminations against the
exercise of the right.
A provision in a state constitution recurring to conditions existing before the adoption of the Fifteenth
Amendment and the continuance of which conditions that amendment prohibited, and making those
[p348] conditions the test of the right to the suffrage, is in conflict with, and void under, the Fifteenth
Amendment.
The establishment of a literacy test for exercising the suffrage is an exercise by the State of a lawful power
vested in it not subject to the supervision of the Federal courts.
Whether a provision in a suffrage statute may be valid under the Federal Constitution if it is so connected
with other provisions that are invalid as to make the whole statute unconstitutional is a question of state
law, but, in the absence of any decision by the state court, this court may, in a case coming from the
Federal courts, determine it for itself.
The suffrage and literacy tests in the amendment of 1910 to the constitution of Oklahoma are so connected
with each other that the unconstitutionality of the former renders the whole amendment invalid.
The facts, which involve the constitutionality under the Fifteenth Amendment of the Constitution of the
United States of the suffrage amendment to the constitution of Oklahoma, known as the Grandfather
Clause, and the responsibility of election officers under § 5508, Rev.Stat., and § 19 of the Penal Code for
preventing people from voting who have the right to vote, are stated in the opinion.
Question - Does the Constitution protect the right of marital privacy against state restrictions on a couple's
Griswold v. CT 1965 ability to be counseled in the use of contraceptives?
Griswold was Director of Planned Parenthood and gave information, instruction and medical advice to
married couples concerning birth control and were convicted under CT law which criminalized the
provision of counseling in the prevention of conception.
SC ruled that a state unconstitutionally interfered with personal privacy in the marriage relationship when
it prohibited anyone, including married couples, from using contraceptives. Together, the First, Third,
Fourth, and Ninth Amendments, create a new constitutional right, the right to privacy in marital relations.
The Connecticut statute conflicts with the exercise of this right and is therefore null and void.
Question - Did the government violate Hamdi's Fifth Amendment right to Due Process by holding him
Hamdi v. Rumsfeld 2003 indefinitely, without access to an attorney, based solely on an Executive Branch declaration that he was an
"enemy combatant" who fought against the United States? Does the separation of powers doctrine require
federal courts to defer to Executive Branch determinations that an American citizen is an "enemy
combatant"?
In the fall of 2001, Yaser Hamdi, an American citizen, was arrested by the United States military in
Afghanistan. He was accused of fighting for the Taliban against the U.S., declared an "enemy combatant,"
and transfered to a military prison in Virginia. Frank Dunham, Jr., a defense attorney in Virginia, filed a
habeas corpus petition in federal district court there, first on his own and then for Hamdi's father, in an
attempt to have Hamdi's detention declared unconstitutional. He argued that the government had violated
Hamdi's Fifth Amendment right to Due Process by holding him indefinitely and not giving him access to
an attorney or a trial. The government countered that the Executive Branch had the right, during wartime,
to declare people who fight against the United States "enemy combatants" and thus restrict their access to
the court system.
The district court ruled for Hamdi, telling the government to release him. On appeal, a Fourth Circuit
Court of Appeals panel reversed, finding that the separation of powers required federal courts to practice
restraint during wartime because "the executive and legislative branches are organized to supervise the
conduct of overseas conflict in a way that the judiciary simply is not." The panel therefore found that it
should defer to the Executive Branch's "enemy combatant" determination.

Yes and no. In an opinion backed by a four-justice plurality and partly joined by two additional justices,
Justice Sandra Day O'Connor wrote that although Congress authorized Hamdi's detention, Fifth
Amendment due process guarantees give a citizen held in the United States as an enemy combatant the
right to contest that detention before a neutral decisionmaker. The plurality rejected the government's
argument that the separation-of-powers prevents the judiciary from hearing Hamdi's challenge. Justice
David H. Souter, joined by Justice Ruth Bader Ginsburg, concurred with the plurality that Hamdi had the
right to challenge in court his status as an enemy combatant. Souter and Ginsburg, however, disagreed
with the plurality's view that Congress authorized Hamdi's detention. Justice Antonin Scalia issued a
dissent joined by Justice John Paul Stevens. Justice Clarence Thomas dissented separately.
Question - Did the Virginia poll tax violate the Equal Protection Clause of the Fourteenth Amendment?
Harper v. Virginia 1965 Annie E. Harper, a resident of Virginia, filed suit alleging that the state's poll tax was unconstitutional.
After a three-judge district court dismissed the complaint, the case went to the Supreme Court. This case
was decided together with Butts v. Harrison.
In a 6-to-3 decision, the Court held that making voter affluence an electoral standard violated the Equal
Protection Clause. The Court found that wealth or fee-paying had no relation to voting qualifications. The
Court also noted that the Equal Protection Clause was not "shackled to the political theory of a particular
era" and that notions of what constituted equal treatment under the Clause were subject to change.
Question - Did Congress, in passing Title II of the 1964 Civil Rights Act, exceed its Commerce Clause
Heart of Atlanta Motel Inc. 1964 powers by depriving motels, such as the Heart of Atlanta, of the right to choose their own customers?
v. US Motel owner in GA challenged the Civil Rights Act of 1964 on the basis he had a local business which
should not be considered interstate commerce as the Act enacted by Congress had the right to ban
discrimination in public accommodations and in employment.
SC ruled that since the business gained most of it income from interstate commerce, thus, the business was
liable.
Question - Does the Fourth Amendment protection against unreasonable searches and seizures require the
Katz v. US 1967 police to obtain a search warrant in order to wiretap a public pay phone?
Acting on a suspicion that Katz was transmitting gambling information over the phone to clients in other
states, Federal agents attached an eavesdropping device to the outside of a public phone booth used by
Katz. Based on recordings of his end of the conversations, Katz was convicted under an eight- count
indictment for the illegal transmission of wagering information from Los Angeles to Boston and Miami.
On appeal, Katz challanged his conviction arguing that the recordings could not be used as evidence
against him. The Court of Appeals rejected this point, noting the absence of a physical intrusion into the
phone booth itself. The Court granted certiorari.
The Court ruled that Katz was entitled to Fourth Amendment protection for his conversations and that a
physical intrusion into the area he occupied was unnecessary to bring the Amendment into play. "The
Fourth Amendment protects people, not places," wrote Justice Potter Stewart for the Court. A concurring
opinion by John Marshall Harlan introduced the idea of a 'reasonable' expectation of Fourth Amendment
protection.
Question - Does a city violate the Fifth Amendment's takings clause if the city takes private property and
Kelo v. City of New London 2004 sells it for private development, with the hopes the development will help the city's bad economy?
New London, a city in Connecticut, used its eminent domain authority to seize private property to sell to
private developers. The city said developing the land would create jobs and increase tax revenues. Kelo
Susette and others whose property was seized sued New London in state court. The property owners
argued the city violated the Fifth Amendment's takings clause, which guaranteed the government will not
take private property for public use without just compensation. Specifically the property owners argued
taking private property to sell to private developers was not public use. The Connecticut Supreme Court
ruled for New London.
. In a 5-4 opinion delivered by Justice John Paul Stevens, the majority held that the city's taking of private
property to sell for private development qualified as a "public use" within the meaning of the takings
clause. The city was not taking the land simply to benefit a certain group of private individuals, but was
following an economic development plan. Such justifications for land takings, the majority argued, should
be given deference. The takings here qualified as "public use" despite the fact that the land was not going
to be used by the public. The Fifth Amendment did not require "literal" public use, the majority said, but
the "broader and more natural interpretation of public use as 'public purpose.'"
Question - Did the President and Congress go beyond their war powers by implementing exclusion and
Korematsu v. US 1944 restricting the rights of Americans of Japanese descent?
FRD placed over 100,000 Japanese Americans in relocation camps after the attack on Pearl Harbor.
SC ruled that relocation was illegal. Questioned the Legality of Japanese Internment during WWII – in
1988 these people were given compensation.
Question - Do the criminal convictions of John Lawrence and Tyron Garner under the Texas "Homosexual
Lawrence v. Texas 2002 Conduct" law, which criminalizes sexual intimacy by same-sex couples, but not identical behavior by
different-sex couples, violate the Fourteenth Amendment guarantee of equal protection of laws? Do their
criminal convictions for adult consensual sexual intimacy in the home violate their vital interests in liberty
and privacy protected by the Due Process Clause of the Fourteenth Amendment? Should Bowers v.
Hardwick, 478 U.S. 186 (1986), be overruled?
Responding to a reported weapons disturbance in a private residence, Houston police entered John
Lawrence's apartment and saw him and another adult man, Tyron Garner, engaging in a private,
consensual sexual act. Lawrence and Garner were arrested and convicted of deviate sexual intercourse in
violation of a Texas statute forbidding two persons of the same sex to engage in certain intimate sexual
conduct. In affirming, the State Court of Appeals held that the statute was not unconstitutional under the
Due Process Clause of the Fourteenth Amendment, with Bowers v. Hardwick, 478 U.S. 186 (1986),
controlling.
In a 6-3 opinion delivered by Justice Anthony M. Kennedy, the Court held that the Texas statute making it
a crime for two persons of the same sex to engage in certain intimate sexual conduct violates the Due
Process Clause. After explaining what it deemed the doubtful and overstated premises of Bowers, the
Court reasoned that the case turned on whether Lawrence and Garner were free as adults to engage in the
private conduct in the exercise of their liberty under the Due Process Clause. "Their right to liberty under
the Due Process Clause gives them the full right to engage in their conduct without intervention of the
government," wrote Justice Kennedy. "The Texas statute furthers no legitimate state interest which can
justify its intrusion into the personal and private life of the individual," continued Justice Kennedy.
Accordingly, the Court overruled Bowers. Justice Sandra Day O'Connor filed an opinion concurring in the
judgment. Justices Clarence Thomas and Antonin Scalia, with whom Chief Justice William H. Rehnquist
and Justices Thomas joined, filed dissents.
Question - Can a plaintiff bring a salary discrimination suit under Title VII of the Civil Rights Act of 1964
Ledbetter v. Goodyear 2006 when the disparate pay is received during the 180-day statutory limitations period, but is the result of
discriminatory pay decisions that occurred outside the limitations period?
Over her nineteen-year career at Goodyear Tire, Lilly Ledbetter was consistently given low rankings in
annual performance-and-salary reviews and low raises relative to other employees. Ledbetter sued
Goodyear for gender discrimination in violation of Title VII of the Civil Rights Act of 1964, alleging that
the company had given her a low salary because of her gender. A jury found for Ledbetter and awarded
her over $3.5 million, which the district judge later reduced to $360,000.
Goodyear appealed, citing a Title VII provision that requires discrimination complaints to made within
180 days of the employer's discriminatory conduct. The jury had examined Ledbetter's entire career for
evidence of discrimination, but Goodyear argued that the jury should only have considered the one annual
salary review that had occurred within the 180-day limitations period before Ledbetter's complaint.
The U.S. Court of Appeals for the Eleventh Circuit reversed the lower court, but without adopting
Goodyear's position entirely. Instead the Circuit Court ruled that the jury could only examine Ledbetter's
career for evidence of discrimination as far back as the last annual salary review before the start of the
180-day limitations period. The Circuit Court ruled that the fact that Ledbetter was getting a low salary
during the 180 days did not justify the evaluation of Goodyear's decisions over Ledbetter's entire career.
Instead, only those annual reviews that could have affected Ledbetter's payment during the 180 days could
be evaluated. The Circuit Court found no evidence of discrimination in those reviews, so it reversed the
District Court and dismissed Ledbetter's complaint.
By a 5-4 vote the Court ruled that Ledbetter's claim was time-barred by Title VII's limitations period. The
opinion by Justice Samuel Alito held that "current effects alone cannot breathe life into prior, uncharged
discrimination." For a timely claim, Ledbetter would have needed to file within 180 days of a
discriminatory salary decision; the Court did not consider it significant that paychecks she received during
the 180 days prior to her claim were affected by the past discrimination. Discriminatory intent is a crucial
element of a Title VII disparate-treatment claim, the Court held, but each instance of Goodyear's
discriminatory intent fell outside the limitations period. The majority noted that the short time limit was
enacted to ensure quick resolution of pay discrimination disputes, which can become more difficult to
defend against as time passes. To adopt Ledbetter's argument would be to allow even "discriminatory pay
decision[s] made 20 years ago" to be the subject of Title VII claims. In dissent, Justice Ruth Bader
Ginsburg called the majority's ruling out of tune with the realities of wage discrimination and "a cramped
interpretation of Title VII, incompatible with the statute's broad remedial purpose." She suggested that "the
Legislature may act to correct this Court's parsimonious reading of Title VII."

Question - Was it an abuse of a court's discretion to allow payment of allocated funds to nonpublic
Lemon v. Kurtzman 1970 religious schools, after such allocations were found unconstitutional by the Supreme Court?
This case was heard concurrently with two others, Earley v. DiCenso (1971) and Robinson v. DiCenso
(1971). The cases involved controversies over laws in Pennsylvania and Rhode Island. In Pennsylvania, a
statute provided financial support for teacher salaries, textbooks, and instructional materials for secular
subjects to non-public schools. The Rhode Island statute provided direct supplemental salary payments to
teachers in non-public elementary schools. Each statute made aid available to "church-related educational
institutions."
Writing for the majority, Chief Justice Burger articulated a three-part test for laws dealing with religious
establishment. To be constitutional, a statute must have "a secular legislative purpose," it must have
principal effects which neither advance nor inhibit religion, and it must not foster "an excessive
government entanglement with religion." The Court found that the subsidization of parochial schools
furthered a process of religious inculcation, and that the "continuing state surveillance" necessary to
enforce the specific provisions of the laws would inevitably entangle the state in religious affairs. The
Court also noted the presence of an unhealthy "divisive political potential" concerning legislation which
appropriates support to religious schools.
Question - Does the New York law violate the liberty protected by due process of the Fourteenth
Lochner v. NY 1905 Amendment?
The state of New York enacted a statute forbidding bakers to work more than 60 hours a week or 10 hours
a day.
Declared unconstitutional a NY act limiting working hours of bakers due to a denial of the 14 th
Amendment rights maintaining that the statute interfered with the freedom of contract, and thus the
Fourteenth Amendment's right to liberty afforded to employer and employee. The Court viewed the statute
as a labor law; the state had no reasonable ground for interfering with liberty by determining the hours of
labor.
Question - 1) Is the prohibition of all protesting within the 36-foot buffer zone around the front of the
Madsen v. Womens Health 1993 clinic an infringement of the First Amendment right to free speech? 2) Is the 36-foot buffer zone along the
Center Inc. back and side of the clinic a breach of the First Amendment right to free speech? 3) Do the limitations
imposed on noise-making constitute a breach of the First Amendment right to free speech? 4) Do the
restrictions placed on the use of images violate the First Amendment right to free speech? 5) Is it a breach
of the First Amendment right to free speech to bar protesters from approaching potential patients when
they are within a 300-foot radius of the clinic? 6) Is it a violation of the First Amendment right to free
speech to prohibit all protesting in a 300-foot radius of clinic staff residences?
Women's Health Center Inc. operated several abortion clinics throughout central Florida, including the
Aware Woman Center for Choice in Melbourne, Florida. In 1992, in response to anti-abortion protesters, a
state court prohibited the protesters from physically abusing those entering or exiting the clinic, or
otherwise interfering with access to the clinic. About 6 months later, Women's Health Center Inc.
expressed a need to broaden the court order. The state court agreed, banning demonstrators from entering a
36-foot buffer- zone around the clinic, making excessive noise, using images visible to patients,
approaching patients within a 300-foot radius of the clinic, and protesting within a 300-foot radius of staff
residences. Petitioner Judy Madsen and her fellow protesters claimed that these restrictions violated their
First Amendment right to free speech, but the Florida Supreme Court disagreed, upholding the court order.
In a majority opinion authored by Chief Justice William H. Rehnquist, the Court found that the state of
Florida could only restrict protesters to the extent necessary to allow the clinic to run and the staff to live
in their homes without interference. Thus, the majority approved of the 36-foot buffer zone around the
front of the clinic because it was essential to allow patients and staff to enter and leave the building freely,
but disapproved of the 36-foot buffer zone along the back and side of the building because it found no
indication that protesting in these areas interfered with the function of the clinic. The Court also
determined that the limitations placed on noise-making were necessary to insure the well-being of the
patients, whereas those placed on images were not because they were easier to ignore. Finally, the Court
concluded that both 300-foot radius rules were too broad, thus restricting the protestors more than was
necessary. Therefore, the decision of the Florida Supreme Court was affirmed in part and reversed in part.
Question - Were the confiscated materials protected by the First Amendment? (May evidence obtained
Mapp v. Ohio 1961 through a search in violation of the Fourth Amendment be admitted in a state criminal proceeding?)
Dollree Mapp was convicted of possessing obscene material after an admittedly illegal police search of her
home for a fugitive. She appealed her conviction on the basis of freedom of expression.
The Court brushed aside the First Amendment issue and declared that "all evidence obtained by searches
and seizures in violation of the Constitution is, by [the Fourth Amendment], inadmissible in a state court."
Mapp had been convicted on the basis of illegally obtained evidence. This was an historic -- and
controversial -- decision. It placed the requirement of excluding illegally obtained evidence from court at
all levels of the government. The decision launched the Court on a troubled course of determining how and
when to apply the exclusionary rule
Supremacy of SC to interpret the Constitution and made Judicial branch equal to the other two – by
Marbury v. Madison 1803 Marshall. Established judicial review.

States cannot tax an institution of the federal government. Hamilton’s national back – MD taxed and clerk
McColloch v. Maryland 1819 said no. Upheld in MD court but SC said no.

Question - Is the sale and distribution of obscene materials by mail protected under the First Amendment's
Miller v. California 1971 freedom of speech guarantee?
Miller, after conducting a mass mailing campaign to advertise the sale of "adult" material, was convicted
of violating a California statute prohibiting the distribution of obscene material. Some unwilling recipients
of Miller's brochures complained to the police, initiating the legal proceedings.
In a 5-to-4 decision, the Court held that obscene materials did not enjoy First Amendment protection. The
Court modified the test for obscenity established in Roth v. United States and Memoirs v. Massachusetts,
holding that "[t]he basic guidelines for the trier of fact must be: (a) whether 'the average person, applying
contemporary community standards' would find that the work, taken as a whole, appeals to the prurient
interest. . . (b) whether the work depicts or describes, in a patently offensive way, sexual conduct
specifically defined by the applicable state law; and (c) whether the work, taken as a whole, lacks serious
literary, artistic, political, or scientific value." The Court rejected the "utterly without redeeming social
value" test of the Memoirs decision.
Question - Does the police practice of interrogating individuals without notifiying them of their right to
Miranda v Arizona 1966 counsel and their protection against self-incrimination violate the Fifth Amendment?
Ernesto Miranda was arrested as a suspect of a kidnapping/rape and maintained his innocence but after two
hours of interrogation he signed a confession. The confession was used as evidence and he was found
guilty even though the police acknowledged that Miranda had not been made aware of his rights given
during the interrogation and did not have access to legal counsel.
SC ruled that the accused must be made of their rights from the beginning The Court held that prosecutors
could not use statements stemming from custodial interrogation of defendants unless they demonstrated
the use of procedural safeguards "effective to secure the privilege against self- incrimination." Hence –
Miranda rights.
Question - 1) Does the First Amendment allow public schools to prohibit students from displaying
Morse v. Frederick 2006
messages promoting the use of illegal drugs at school-supervised events?

2) Does a school official have qualified immunity from a damages lawsuit under 42 U.S.C. 1983 when, in
accordance with school policy, she disciplines a student for displaying a banner with a drug reference at a
school-supervised event?

At a school-supervised event, Joseph Frederick held up a banner with the message "Bong Hits 4 Jesus," a
slang reference to marijuana smoking. Principal Deborah Morse took away the banner and suspended
Frederick for ten days. She justified her actions by citing the school's policy against the display of material
that promotes the use of illegal drugs. Frederick sued under 42 U.S.C. 1983, the federal civil rights statute,
alleging a violation of his First Amendment right to freedom of speech. The District Court found no
constitutional violation and ruled in favor of Morse. The court held that even if there were a violation, the
principal had qualified immunity from lawsuit. The U.S. Court of Appeals for the Ninth Circuit reversed.
The Ninth Circuit cited Tinker v. Des Moines Independent Community School District, which extended
First Amendment protection to student speech except where the speech would cause a disturbance.
Because Frederick was punished for his message rather than for any disturbance, the Circuit Court ruled,
the punishment was unconstitutional. Furthermore, the principal had no qualified immunity, because any
reasonable principal would have known that Morse's actions were unlawful.
Yes and not reached. The Court reversed the Ninth Circuit by a 5-4 vote, ruling that school officials can
prohibit students from displaying messages that promote illegal drug use. Chief Justice John Roberts's
majority opinion held that although students do have some right to political speech even while in school,
this right does not extend to pro-drug messages that may undermine the school's important mission to
discourage drug use. The majority held that Frederick's message, though "cryptic," was reasonably
interpreted as promoting marijuana use - equivalent to "[Take] bong hits" or "bong hits [are a good thing]."
In ruling for Morse, the Court affirmed that the speech rights of public school students are not as extensive
as those adults normally enjoy, and that the highly protective standard set by Tinker would not always be
applied. In concurring opinions, Justice Thomas expressed his view that the right to free speech does not
apply to students and his wish to see Tinker overturned altogether, while Justice Alito stressed that the
decision applied only to pro-drug messages and not to broader political speech. The dissent conceded that
the principal should have had immunity from the lawsuit, but argued that the majority opinion was "[...]
deaf to the constitutional imperative to permit unfettered debate, even among high-school students [...]."

National Federation of 2011


Independent Businesses v.
Sebelius

Question - Did Alabama's libel law, by not requiring Sullivan to prove that an advertisement personally
NY Times v. Sullivan 1964 harmed him and dismissing the same as untruthful due to factual errors, unconstitutionally infringe on the
First Amendment's freedom of speech and freedom of press protections?
Full-page ad in NYT which alleged that the arrest of MLK Jr. for perjury in Alabama was part of a
campaign to destroy MLK’s efforts to integrate public facilities and encourage blacks to vote for LB
Sullivan, the Commissioner sued the paper saying that the allegations defamed him personally. Court
ruled that the 1st Amendment guarantee of freedom of the press protected the press from libel suits for
defamatory reports on public officials unless the officials proved that the reports were made of actual
malice.
SC defined malice as ‘with knowledge that they are false or with reckless disregard of their truth or
falsity”. Case found in favor of NYT under 1st.
Question - Did the Nixon administration's efforts to prevent the publication of what it termed "classified
NY Times v. US 1970 information" violate the First Amendment?
In what became known as the "Pentagon Papers Case," the Nixon Administration attempted to prevent the
New York Times and Washington Post from publishing materials belonging to a classified Defense
Department study regarding the history of United States activities in Vietnam. The President argued that
prior restraint was necessary to protect national security. This case was decided together with United
States v. Washington Post Co.
Yes. In its per curiam opinion the Court held that the government did not overcome the "heavy
presumption against" prior restraint of the press in this case. Justices Black and Douglas argued that the
vague word "security" should not be used "to abrogate the fundamental law embodied in the First
Amendment." Justice Brennan reasoned that since publication would not cause an inevitable, direct, and
immediate event imperiling the safety of American forces, prior restraint was unjustified.
Question - Did the requirement constitute a property taking in violation of the Fifth and Fourteenth
Nolan v. Ca. Coastal 1986 Amendments?
Commission The California Coastal Commission required owners of beachfront property wishing to obtain a building
permit to maintain a pathway on their property open to the public.
The Court agreed that a legitimate interest may be served by maintaining a "continuous strip of publicly
accessible beach along the coast." However, reasoned Justice Scalia, if California wished to use its power
of eminent domain to do so, it must provide just compensation to the Nollans and other beachfront
property owners for the public use of their land.
Question - Is Louisiana's law mandating racial segregation on its trains an unconstitutional infringement on
Plessy v. Ferguson 1896 both the privileges and immunities and the equal protection clauses of the Fourteenth Amendment?
Plessy (1/8 black) was arrested on Louisiana train for sitting in White-only car and asked for an appeal to
the 14th Amendment of due process.
SC said so long as the facilities were equal then it was legal to racially separate. Beginning of Jim Crow
laws in the South and impetus of segregation.
Question - Can a state require women who want an abortion to obtain informed consent, wait 24 hours,
Planned Parenthood v. 1991 and, if minors, obtain parental consent, without violating their right to abortions as guaranteed by Roe v.
Casey Wade?
The Pennsylvania legislature amended its abortion control law in 1988 and 1989. Among the new
provisions, the law required informed consent and a 24 hour waiting period prior to the procedure. A
minor seeking an abortion required the consent of one parent (the law allows for a judicial bypass
procedure). A married woman seeking an abortion had to indicate that she notified her husband of her
intention to abort the fetus. These provisions were challenged by several abortion clinics and physicians. A
federal appeals court upheld all the provisions except for the husband notification requirement.
In a bitter, 5-to-4 decision, the Court again reaffirmed Roe, but it upheld most of the Pennsylvania
provisions. For the first time, the justices imposed a new standard to determine the validity of laws
restricting abortions. The new standard asks whether a state abortion regulation has the purpose or effect
of imposing an "undue burden," which is defined as a "substantial obstacle in the path of a woman seeking
an abortion before the fetus attains viability." Under this standard, the only provision to fail the undue-
burden test was the husband notification requirement. The opinion for the Court was unique: It was crafted
and authored by three justices.
Question - Did the University of California violate the Fourteenth Amendment's equal protection clause,
Regents of U of C v. Bakke and the Civil Rights Act of 1964, by practicing an affirmative action policy that resulted in the repeated
rejection of Bakke's application for admission to its medical school?
Allan Bakke, a thirty-five-year-old white man, had twice applied for admission to the University of
California Medical School at Davis. He was rejected both times. The school reserved sixteen places in
each entering class of one hundred for "qualified" minorities, as part of the university's affirmative action
program, in an effort to redress longstanding, unfair minority exclusions from the medical profession.
Bakke's qualifications (college GPA and test scores) exceeded those of any of the minority students
admitted in the two years Bakke's applications were rejected.
SC said the university can use special criteria to determine which students gained acceptance so long as it
did not use a quota system. There was no single majority opinion. Four of the justices contended that any
racial quota system supported by government violated the Civil Rights Act of 1964. Justice Lewis F.
Powell, Jr., agreed, casting the deciding vote ordering the medical school to admit Bakke. However, in his
opinion, Powell argued that the rigid use of racial quotas as employed at the school violated the equal
protection clause of the Fourteenth Amendment. The remaining four justices held that the use of race as a
criterion in admissions decisions in higher education was constitutionally permissible. Powell joined that
opinion as well, contending that the use of race was permissible as one of several admission criteria. So,
the Court managed to minimize white opposition to the goal of equality (by finding for Bakke) while
extending gains for racial minorities through affirmative action.
Question - Did certain provisions of the 1996 Communications Decency Act violate the First and Fifth
Reno v. ACLU 1996 Amendments by being overly broad and vague in their definitions of the types of internet communications
which they criminalized?
Several litigants challenged the constitutionality of two provisions in the 1996 Communications Decency
Act. Intended to protect minors from unsuitable internet material, the Act criminalized the intentional
transmission of "obscene or indecent" messages as well as the transmission of information which depicts
or describes "sexual or excretory activities or organs" in a manner deemed "offensive" by community
standards. After being enjoined by a District Court from enforcing the above provisions, except for the one
concerning obscenity and its inherent protection against child pornography, Attorney General Janet Reno
appealed directly to the Supreme Court as provided for by the Act's special review provisions.
Yes. The Court held that the Act violated the First Amendment because its regulations amounted to a
content-based blanket restriction of free speech. The Act failed to clearly define "indecent"
communications, limit its restrictions to particular times or individuals (by showing that it would not
impact adults), provide supportive statements from an authority on the unique nature of internet
communications, or conclusively demonstrate that the transmission of "offensive" material is devoid of
any social value. The Court added that since the First Amendment distinguishes between "indecent" and
"obscene" sexual expressions, protecting only the former, the Act could be saved from facial overbreadth
challenges if it dropped the words "or indecent" from its text. The Court refused to address any Fifth
Amendment issues.
Question - Did Alabama's apportionment scheme violate the Fourteenth Amendment's Equal Protection
Reynolds v. Sims 1963 Clause by mandating at least one representative per county and creating as many senatorial districts as
there were senators, regardless of population variances?
In 1961, M.O. Sims, David J. Vann (Vann v. Baggett), John McConnell (McConnell v. Baggett), and other
voters from Jefferson County, Alabama, challenged the apportionment of the state legislature. The
Alabama Constitution prescribed that each county was entitled to at least one representative and that there
were to be as many senatorial districts as there were senators. Population variance ratios of as great as 41-
to-1 existed in the Senate.
In an 8-to-1 decision, the Court upheld the challenge to the Alabama system, holding that Equal Protection
Clause demanded "no less than substantially equal state legislative representation for all citizens...."
Noting that the right to direct representation was "a bedrock of our political system," the Court held that
both houses of bicameral state legislatures had to be apportioned on a population basis. States were
required to "honest and good faith" efforts to construct districts as nearly of equal population as
practicable.
Question - Does the Constitution embrace a woman's right to terminate her pregnancy by abortion?
Roe v. Wade 1973 With increase in abortions during the 60s and 70s the SC struck down Texas law that made it illegal to
perform abortions unless the women’s life was at risk. ‘ Jane Roe’ an unmarried mother wanted to
terminate her pregnancy but did not meet the necessary requirements.
SC sided with Roe in that a women has a constitutional right to privacy that extended to cover a decision
whether or not to terminate her pregnancy. The Court held that a woman's right to an abortion fell within
the right to privacy (recognized in Griswold v. Connecticut) protected by the Fourteenth Amendment. The
decision gave a woman total autonomy over the pregnancy during the first trimester and defined different
levels of state interest for the second and third trimesters. As a result, the laws of 46 states were affected
by the Court's ruling.
Question - Does Amendment 2 of Colorado's State Constitution, forbidding the extension of official
Romer v. Evans 1995 protections to those who suffer discrimination due to their sexual orientation, violate the Fourteenth
Amendment's Equal Protection Clause?
Colorado voters adopted Amendment 2 to their State Constitution precluding any judicial, legislative, or
executive action designed to protect persons from discrimination based on their "homosexual, lesbian, or
bisexual orientation, conduct, practices or relationships." Following a legal challenge by homosexual and
other aggrieved parties, the state trial court entered a permanent injunction enjoining Amendment 2's
enforcement. The Colorado Supreme Court affirmed on appeal.
In a 6-to-3 decision, the Court held that Amendment 2 of the Colorado State Constitution violated the
equal protection clause. Amendment 2 singled out homosexual and bisexual persons, imposing on them a
broad disability by denying them the right to seek and receive specific legal protection from
discrimination. In his opinion for the Court, Justice Anthony Kennedy noted that oftentimes a law will be
sustained under the equal protection clause, even if it seems to disadvantage a specific group, so long as it
can be shown to "advance a legitimate government interest." Amendment 2, by depriving persons of equal
protection under the law due to their sexual orientation failed to advance such a legitimate interest. Justice
Kennedy concluded: "If the constitutional conception of 'equal protection of the laws' means anything, it
must at the very least mean that a bare desire to harm a politically unpopular group cannot constitute a
legitimate governmental interest."
Question - Are Schenck's actions (words, expression) protected by the free speech clause of the First
Schenck v. US 1919 Amendment?
Freedom of speech writing during wartime – During World War I, Schenck mailed circulars to draftees.
The circulars suggested that the draft was a monstrous wrong motivated by the capitalist system. The
circulars urged "Do not submit to intimidation" but advised only peaceful action such as petitioning to
repeal the Conscription Act. Schenck was charged with conspiracy to violate the Espionage Act by
attempting to cause insubordination in the military and to obstruct recruitment.
Justice Oliver Wendell Holmes rejected the argument that the pamphlet was protected under the 1 st
amendment and free speech and that it could be suppressed if there was a ‘clear and present danger’ to the
wellbeing of the country. "The question in every case is whether the words used are used in such
circumstances and are of such a nature as to create a clear and present danger that they will bring about the
substantive evils that Congress has a right to prevent." During wartime, utterances tolerable in peacetime
can be punished.
Question - Was Dred Scott free or slave?
Dred Scott v. Sanford 1857 Dred Scott was a slave and traveled with his ‘owner’ to free state (IL). Upon returning to Missouri (slave
state) Scott sued for his freedom. SC decided it had no power to forbid slavery in the territories that are
deemed free and that slaves were not citizens and did not have judiciary rights. Case was never about
slavery but about the expansion of it in the territories and if slavers were deemed as property. Dred Scott
was a slave. Under Articles III and IV, argued Taney, no one but a citizen of the United States could be a
citizen of a state, and that only Congress could confer national citizenship. Taney reached the conclusion
that no person descended from an American slave had ever been a citizen for Article III purposes. The
Court then held the Missouri Compromise unconstitutional, hoping to end the slavery question once and
for all.
Question - Did the North Carolina residents' claim, that the State created a racially gerrymandered district,
Shaw v. Reno 1992 raise a valid constitutional issue under the Fourteenth Amendment's Equal Protection Clause
The U.S. Attorney General rejected a North Carolina congressional reapportionment plan because the plan
created only one black-majority district. North Carolina submitted a second plan creating two black-
majority districts. One of these districts was, in parts, no wider than the interstate road along which it
stretched. Five North Carolina residents challenged the constitutionality of this unusually shaped district,
alleging that its only purpose was to secure the election of additional black representatives. After a three-
judge District Court ruled that they failed to state a constitutional claim, the residents appealed and the
Supreme Court granted certiorari.
The Court held that although North Carolina's reapportionment plan was racially neutral on its face, the
resulting district shape was bizarre enough to suggest that it constituted an effort to separate voters into
different districts based on race. The unusual district, while perhaps created by noble intentions, seemed to
exceed what was reasonably necessary to avoid racial imbalances. After concluding that the residents'
claim did give rise to an equal protection challenge, the Court remanded - adding that in the absence of
contradictory evidence, the District Court would have to decide whether or not some compelling
governmental interest justified North Carolina's plan.
Question - Did denying blacks the right to vote in primary elections violate the Fifteenth Amendment?
Smith v. Allwright 1943 A resolution of the Democratic Party of Texas, a group that the Texas Supreme Court had deemed a
"voluntary association," allowed only whites to participate in Democratic primary elections. S.S. Allwright
was a county election official; he denied Lonnie E. Smith, a black man, the right to vote in the 1940 Texas
Democratic primary.
The Court overruled its decision in Grovey v. Townsend (1935) and found the restrictions against blacks
unconstitutional. Even though the Democratic Party was a voluntary organization, the fact that Texas
statutes governed the selection of county-level party leaders, the party conducted primary elections under
state statutory authority, and state courts were given exclusive original jurisdiction over contested
elections, guaranteed for blacks the right to vote in primaries. Allwright engaged in state action abridging
Smith's right to vote because of his race. A state cannot "permit a private organization to practice racial
discrimination" in elections, argued Justice Reed. (The Court's decision in this matter was amended on
June 12, 1944.)
Question - Were federal courts constitutionally authorized to oversee and produce remedies for state-
Swann v. Charlotte 1970 imposed segregation?
Mechlenburg Bd of Ed. After the Supreme Court's decision in 1954 in Brown v. Board of Education, little progress had been made
in desegregating public schools. One example was the Charlotte-Mecklenburg, North Carolina, system in
which approximately 14,000 black students attended schools that were either totally black or more than 99
percent black. Lower courts had experimented with a number of possible solutions when the case reached
the Supreme Court.
In a unanimous decision, the Court held that once violations of previous mandates directed at
desegregating schools had occurred, the scope of district courts' equitable powers to remedy past wrongs
were broad and flexible. The Court ruled that 1) remedial plans were to be judged by their effectiveness,
and the use of mathematical ratios or quotas were legitimate "starting points" for solutions; 2)
predominantly or exclusively black schools required close scrutiny by courts; 3) non-contiguous
attendance zones, as interim corrective measures, were within the courts' remedial powers; and 4) no rigid
guidelines could be established concerning busing of students to particular schools.
Question - Is the desecration of an American flag, by burning or otherwise, a form of speech that is
Texas v. Johnson 1988 protected under the First Amendment?
In 1984, in front of the Dallas City Hall, Gregory Lee Johnson burned an American flag as a means of
protest against Reagan administration policies. Johnson was tried and convicted under a Texas law
outlawing flag desecration. He was sentenced to one year in jail and assessed a $2,000 fine. After the
Texas Court of Criminal Appeals reversed the conviction, the case went to the Supreme Court.
In a 5-to-4 decision, the Court held that Johnson's burning of a flag was protected expression under the
First Amendment. The Court found that Johnson's actions fell into the category of expressive conduct and
had a distinctively political nature. The fact that an audience takes offense to certain ideas or expression,
the Court found, does not justify prohibitions of speech. The Court also held that state officials did not
have the authority to designate symbols to be used to communicate only limited sets of messages, noting
that "[i]f there is a bedrock principle underlying the First Amendment, it is that the Government may not
prohibit the expression of an idea simply because society finds the idea itself offensive or disagreeable."
Question - Does a prohibition against the wearing of armbands in public school, as a form of symbolic
Tinker v. Des Moines 1968 protest, violate the students' freedom of speech protections guaranteed by the First Amendment?
In December 1965, a group of students in Des Moines held a meeting in the home of 16-year-old
Christopher Eckhardt to plan a public showing of their support for a truce in the Vietnam war. They
decided to wear black armbands throughout the holiday season and to fast on December 16 and New
Year’s Eve. The principals of the Des Moines school learned of the plan and met on December 14 to create
a policy that stated that any student wearing an armband would be asked to remove it, with refusal to do so
resulting in suspension. On December 16, Mary Beth Tinker and Christopher Eckhardt wore their
armbands to school and were sent home. The following day, John Tinker did the same with the same
result. The students did not return to school until after New Year’s Day, the planned end of the protest.
Through their parents, the students sued the school district for violating the students’ right of expression
and sought an injunction to prevent the school district from disciplining the students. The district court
dismissed the case and held that the school district’s actions were reasonable to uphold school discipline.
The U.S. Court of Appeals for the Eighth Circuit affirmed the decision without opinion.
Yes. Justice Abe Fortas delivered the opinion of the 7-2 majority. The Supreme Court held that the
armbands represented pure speech that is entirely separate from the actions or conduct of those
participating in it. The Court also held that the students did not lose their First Amendment rights to
freedom of speech when they stepped onto school property. In order to justify the suppression of speech,
the school officials must be able to prove that the conduct in question would “materially and substantially
interfere” with the operation of the school. In this case, the school district’s actions evidently stemmed
from a fear of possible disruption rather than any actual interference.
In his concurring opinion, Justice Potter Stewart wrote that children are not necessarily guaranteed the full
extent of First Amendment rights. Justice Byron R. White wrote a separate concurring opinion in which he
noted that the majority’s opinion relies on a distinction between communication through words and
communication through action.
Justice Hugo L. Black wrote a dissenting opinion in which he argued that the First Amendment does not
provide the right to express any opinion at any time. Because the appearance of the armbands distracted
students from their work, they detracted from the ability of the school officials to perform their duties, so
the school district was well within its rights to discipline the students. In his separate dissent, Justice John
M. Harlan argued that school officials should be afforded wide authority to maintain order unless their
actions can be proven to stem from a motivation other than a legitimate school interest

Question - Is the President's right to safeguard certain information, using his "executive privilege"
US v. Nixon 1974 confidentiality power, entirely immune from judicial review?
A grand jury returned indictments against seven of President Richard Nixon's closest aides in the
Watergate affair. The special prosecutor appointed by Nixon and the defendants sought audio tapes of
conversations recorded by Nixon in the Oval Office. Nixon asserted that he was immune from the
subpoena claiming "executive privilege," which is the right to withhold information from other
government branches to preserve confidential communications within the executive branch or to secure the
national interest. Decided together with Nixon v. United States.
SC rejected Nixon’s claim to an absolutely unqualified privilege against any judicial process. The Court
held that neither the doctrine of separation of powers, nor the generalized need for confidentiality of high-
level communications, without more, can sustain an absolute, unqualified, presidential privilege. The
Court granted that there was a limited executive privilege in areas of military or diplomatic affairs, but
gave preference to "the fundamental demands of due process of law in the fair administration of justice."
Therefore, the president must obey the subpoena and produce the tapes and documents. Nixon resigned
shortly after the release of the tapes.
Question - Was the act a legitimate exercise of Congress's power to regulate interstate commerce?
US v. Darby 1940 In 1938, Congress passed the Fair Labor Standards Act to regulate many aspects of employment including
minimum wages, maximum weekly hours, and child labor. Corporations which engaged in interstate
commerce or produced goods which were sold in other states were punished for violating the statute.
The unanimous Court affirmed the right of Congress to exercise "to its utmost extent" the powers reserved
for it in the Commerce Clause. Relying heavily on the Court's decision in Gibbons v. Ogden (1824),
Justice Stone argued that the "motive and purpose of a regulation of interstate commerce are matters for
the legislative judgment . . . over which the courts are given no control." Congress acted with proper
authority in outlawing substandard labor conditions since they have a significant impact on interstate
commerce.
Question - Does the executive branch’s agreement with the lower court that the act is unconstitutional
US v. Windsor 2012 deprive the Supreme Court of jurisdiction to decide the case?
Does the Bipartisan Legal Advisory Group of the House of Representatives have standing in the case?
Does the Defense of Marriage Act, which defines the term “marriage” under federal law as a “legal union
between one man and one woman” deprive same-sex couples who are legally married under state laws of
their Fifth Amendment rights to equal protection under federal law?

The Defense of Marriage Act (DOMA), enacted in 1996, states that, for the purposes of federal law, the
words “marriage” and “spouse” refer to legal unions between one man and one woman. Since that time,
some states have authorized same-sex marriage. In other cases regarding the DOMA, federal courts have
ruled it unconstitutional under the Fifth Amendment, but the courts have disagreed on the rationale.
Edith Windsor is the widow and sole executor of the estate of her late spouse, Thea Clara Spyer, who died
in 2009. The two were married in Toronto, Canada, in 2007, and their marriage was recognized by New
York state law. Thea Syper left her estate to her spouse, and because their marriage was not recognized by
federal law, the government imposed $363,000 in taxes. Had their marriage been recognized, the estate
would have qualified for a marital exemption, and no taxes would have been imposed.
On November 9, 2010 Windsor filed suit in district court seeking a declaration that the Defense of
Marriage Act was unconstitutional. At the time the suit was filed, the government’s position was that
DOMA must be defended. On February 23, 2011, the President and the Attorney General announced that
they would not defend DOMA. On April 18, 2011, the Bipartisan Legal Advisory Group of the House of
Representatives filed a petition to intervene in defense of DOMA and motioned to dismiss the case. The
district court denied the motion, and later held that DOMA was unconstitutional. The U.S. Court of
Appeals for the Second Circuit affirmed.
Yes, unanswered, yes. Justice Anthony M. Kennedy delivered the opinion of the 5-4 majority. The
Supreme Court held that the United States Government, despite the executive branch’s agreement
regarding DOMA’s unconstitutionality, retains a significant enough stake in the issue to support Supreme
Court’s jurisdiction. Because the judgment in question orders the U.S. Treasury to refund tax money, the
Government stands to suffer a real economic injury and therefore maintains standing in the case. The
Bipartisan Legal Advisory Group (BLAG) presented substantial arguments for the constitutionality of
DOMA that reflected an actual controversy under Article III, which allowed the Supreme Court to address
the case without needing to decide whether BLAG would have had standing before a lower court. The
Court also held that states have the authority to define marital relationships and that DOMA goes against
legislative and historical precedent by undermining that authority. The result is that DOMA denies same-
sex couples the rights that come from federal recognition of marriage, which are available to other couples
with legal marriages under state law. The Court held that the purpose and effect of DOMA is to impose a
“disadvantage, a separate status, and so a stigma” on same-sex couples in violation of the Fifth
Amendment’s guarantee of equal protection.
Chief Justice John G. Roberts wrote a dissent in which he argued that the Court lacked the jurisdiction to
review the case and that interests in uniformity and stability justified Congress’ enactment of DOMA. He
also argued that the majority’s opinion did not address the issue of state definitions of marriage affecting
same-sex couples. In his separate dissent, Justice Antonin Scalia wrote that the Supreme Court had neither
the jurisdiction to review the case nor the power to invalidate democratically enacted legislation. He
argued that the majority’s opinion wrongly asserted the supremacy of the Supreme Court as the final
arbiter of government. However, the majority opinion did not address the issue of whether or not the Equal
Protection Clause required laws restricting the definition of marriage to be reviewed under a rational basis
or strict scrutiny standard. He also argued that the majority misconstrued DOMA’s insidious intent and
should not rule based on that presumption. Justice Clarence Thomas and Chief Justice Roberts joined in
the dissent. Justice Samuel A. Alito, Jr. also wrote a separate dissent in which he argued that the United
States Government did not have standing in the case because the executive branch declined to defend the
statute, but that BLAG did have standing because it chose to defend the otherwise undefended statute. He
also argued that the Constitution does not guarantee the right to enter into a same-sex marriage because
that right is not “deeply rooted in this Nation’s history and tradition.” Instead, the issue of the definition of
marriage is left to the people to decide, a decision in which DOMA does not interfere. Justice Clarence
Thomas partially joined in the dissent.

Question - Does the state of Georgia have the authority to regulate the intercourse between citizens of its
Worcester v. Georgia 1832 state and members of the Cherokee Nation?
Marshall invalidated/cancelled GA law concerning entry into the Cherokee nation. In September 1831,
Samuel A. Worcester and others, all non-Native Americans, were indicted in the supreme court for the
county of Gwinnett in the state of Georgia for "residing within the limits of the Cherokee nation without a
license" and "without having taken the oath to support and defend the constitution and laws of the state of
Georgia." They were indicted under an 1830 act of the Georgia legislature entitled "an act to prevent the
exercise of assumed and arbitrary power by all persons, under pretext of authority from the Cherokee
Indians." Among other things, Worcester argued that the state could not maintain the prosecution because
the statute violated the Constitution, treaties between the United States and the Cherokee nation, and an act
of Congress entitled "an act to regulate trade and intercourse with the Indian tribes." Worcester was
convicted and sentenced to "hard labour in the penitentiary for four years." The U.S. Supreme Court
received the case on a writ of error.
In an opinion delivered by Chief Justice John Marshall, the Court held that the Georgia act, under which
Worcester was prosecuted, violated the Constitution, treaties, and laws of the United States. Noting that
the "treaties and laws of the United States contemplate the Indian territory as completely separated from
that of the states; and provide that all intercourse with them shall be carried on exclusively by the
government of the union," Chief Justice Marshall argued, "The Cherokee nation, then, is a distinct
community occupying its own territory in which the laws of Georgia can have no force. The whole
intercourse between the United States and this nation, is, by our constitution and laws, vested in the
government of the United States." The Georgia act thus interfered with the federal government's authority
and was unconstitutional. Justice Henry Baldwin dissented for procedural reasons and on the merits.
States had no rights to control any aspect of the Cherokee nation as it would fall under the powers of the
federal government. Another example of Federal supremacy over the states. Attempt to protect the rights
of NA Indians.

You might also like